Ask your own question, for FREE!
Mathematics 29 Online
OpenStudy (phoenixfire):

Sequence: \(x_1=1\), recursive definition: \(x_{n+1}=\sqrt{1+x_n} \) for all n>=1 Use induction to prove \(x_n\) is bounded above by 2. I tried: Bounded above assumption: \(x_n \le 2\) Let k=n=1, then \[x_1 \le 2\] \[1 \le 2\] Induction: Let k=n+1, then \[x_{n+1} \le 2\] \[\sqrt{1+x_n} \le 2\] \[x_n \le 3\] And since we assume \(x_n \le 2\) holds true for all n, then \(2 \le 3\) is true and the sequence is bounded above by 2.

OpenStudy (perl):

hmmm

OpenStudy (perl):

for the induction step, you want to assume that xn is bounded above by 2 , for n = k

OpenStudy (perl):

you cant assume that x_n+1 <=2 , that is what you want to prove

zepdrix (zepdrix):

\[\Large\rm x_1=1\]\[\Large\rm x_2=\sqrt{1+1}=\sqrt2\]\[\Large\rm x_3=\sqrt{1+\sqrt2}\]\[\Large\rm x_4=\sqrt{1+\sqrt{1+\sqrt2}}\]\[\Large\rm x_n=\sqrt{1+\sqrt{1+\sqrt{1+...+\sqrt{1+\sqrt{2}}}}}\]Wow this is a weird recursion XD I'm not even sure if I wrote the x_n correctly lol

OpenStudy (perl):

|dw:1402975510227:dw|

Can't find your answer? Make a FREE account and ask your own questions, OR help others and earn volunteer hours!

Join our real-time social learning platform and learn together with your friends!
Latest Questions
YoungBlood: STOP THE YELLOW NAME TAG SLANDER!
2 hours ago 11 Replies 2 Medals
Bubblezz: Art for @euphoriiic
5 hours ago 23 Replies 3 Medals
ilovemybf: i need more drawing ideas
6 hours ago 15 Replies 1 Medal
toga: what is a Mayuri
10 hours ago 3 Replies 1 Medal
Midnight97: Here is a beat I made let me know what y'all think
10 hours ago 24 Replies 2 Medals
toga: who thinks that there should be more titles
11 hours ago 5 Replies 0 Medals
Can't find your answer? Make a FREE account and ask your own questions, OR help others and earn volunteer hours!

Join our real-time social learning platform and learn together with your friends!